5
$\begingroup$

Let $f$ be a continuous map ${\mathbb R}^2 \to {\mathbb R}$. For $y\in {\mathbb R}$, denote by $P_y$ the preimage set $\lbrace (x_1,x_2) \in {\mathbb R}^2 | f(x_1,x_2)=y \rbrace$.

Is it true that

(1) At least one $P_y$ is uncountable ?

(2) At least one $P_y$ has the same cardinality as $\mathbb R$.

Some easy remarks :

  • (2) is stronger than (1).

  • (2) follows from (1) if we assume the GCH.

  • If there is a point $(x_0,y_0)$ such that the partial derivatives $\frac{\partial f}{\partial x}(x_0,y_0)$ and $\frac{\partial f}{\partial y}(x_0,y_0)$ exist and one of them is nonzero, then (2) (and hence (1)) follows from the implicit function theorem.

3 Answers 3

8

Consider the restriction of $f$ to $\mathbb{R}_x=\mathbb{R} \times \{x\}$ for fixed $x$. If its image is a point, we're done. Otherwise, its image is an interval in $\mathbb{R}$, and that interval contains a subinterval with rational endpoints. Since there are only countably many such intervals, there must be one that's contained in the images of uncountably many $\mathbb{R}_x$; then any point in that interval has uncountable preimage.

7

It’s well-known that every uncountable closed subset of $\Bbb R^n$ (indeed, every uncountable Borel set) has cardinality $2^\omega$, so (1) and (2) are equivalent, since every $P_y$ is closed. In fact a strong form of (2) is true.

(2) is certainly true if $f$ is constant. If not, $\operatorname{ran}f$ contains an open interval $(a,b)$. For each $y\in(a,b)$, $\Bbb R^2\setminus P_y$ must be disconnected. But for any countable set $S\subseteq\Bbb R^2$, $\Bbb R^2\setminus S$ is arcwise connected and therefore connected, so $|P_y|=2^\omega$.

To see that $\Bbb R^2\setminus S$ is arcwise connected, fix $p,q\in\Bbb R^2\setminus S$ with $p\ne q$. There are $2^\omega$ straight lines through $p$, so most of them miss $S$, and similarly for $q$. Thus, there are straight lines through $p$ and $q$ that miss $S$ and intersect.

  • 0
    Nice proof that uses connectivity in every possible way.2012-06-16
4

(1) Deleted my answer for this part.

(2) If you have that there exists a $P_y$ which is uncountable, then $P_y$ has the cardinality of $\mathbb{R}$ even without the continuum hypothesis. This is because since $f$ is continuous, $P_y$ is a closed set since $y$ is closed. By the Cantor Bedixson theorem, it can be written as a union of a countable set and a perfect set. Perfect sets have cardinality of the continuum. (I guess they are called perfect because they are not counterexamples to the continuum hypothesis.)

  • 0
    Aha, you were right to edit your original, mistaken claim. It is shown [here](http://math.stackexchange.com/questions/116350/continuous-injective-map-f-mathbbr3-to-mathbbr) that there is no continuous injective function ${\mathbb R}^2 \to {\mathbb R}$.2012-06-16